Đến nội dung

nhungvienkimcuong nội dung

Có 204 mục bởi nhungvienkimcuong (Tìm giới hạn từ 08-05-2020)



Sắp theo                Sắp xếp  

#742347 $\frac{a+b+c}{3}-\sqrt[3]{abc}...

Đã gửi bởi nhungvienkimcuong on 04-12-2023 - 18:26 trong Bất đẳng thức và cực trị

cho a,b,c là các số thực dương. CMR:
$\frac{a+b+c}{3}-\sqrt[3]{abc}\leq max {(\sqrt{a}-\sqrt{b})^{2},(\sqrt{b}-\sqrt{c})^{2},(\sqrt{c}-\sqrt{a})^{2}}$

                                                                                    (đề chọn đội tuyển dự thi Toán quốc tế, Mỹ năm 2000)

Bài này khá lỏng, dễ thấy 

\[\max \left( (\sqrt{a}-\sqrt{b})^{2},(\sqrt{b}-\sqrt{c})^{2},(\sqrt{c}-\sqrt{a})^{2}\right )\ge \frac{\sum (\sqrt{a}-\sqrt{b})^{2}}{3}=\frac{2}{3}\left(\sum a-\sum \sqrt{ab}\right).\]

Như vậy ta cần chứng minh

\[\frac{1}{3}\sum a-\sqrt[3]{abc}\le\frac{2}{3}\left(\sum a-\sum \sqrt{ab}\right)\iff \sum a+3\sqrt[3]{abc}\ge 2\sum\sqrt{ab}.\]

Tuy nhiên theo bất đẳng thức Schur thì

\[\sum a+3\sqrt[3]{abc}\ge \sum\sqrt[3]{ab}(\sqrt[3]{a}+\sqrt[3]{b})\ge \sum\sqrt[3]{ab}\cdot 2\sqrt{\sqrt[3]{ab}}=2\sum \sqrt{ab}.\]




#742329 $\int_{0}^{1}(f'(x))^{2}=\in...

Đã gửi bởi nhungvienkimcuong on 02-12-2023 - 17:28 trong Tài liệu và chuyên đề Giải tích

Mọi người giúp mình bài này với ạ:

Cho hàm số f(x) có đạo hàm liên tục trên [0;1] thỏa mãn $f(1)=0$ và $\int_{0}^{1}(f'(x))^{2}=\int_{0}^{1}(x+1)e^{x}f(x)dx=\frac{e^{2}-1}{4}$. Tính $\int_{0}^{1}f(x)dx$

Ta có

\[\int_{0}^{1}(x+1)e^{x}f(x)\mathrm{d}x=xe^xf(x)\Big|_0^1-\int_{0}^{1}xe^xf'(x)\mathrm{d}x,\]

kết hợp với giả thiết suy ra $\int_{0}^{1}xe^xf'(x)\mathrm{d}x=\frac{1-e^2}{4}$. Theo bất đẳng thức tích phân thì

\[\left(\int_{0}^{1}xe^xf'(x)\mathrm{d}x\right)^2\le \int_{0}^{1}\left(xe^x \right )^2\mathrm{d}x\cdot \int_{0}^{1}\left(f'(x) \right )^2\mathrm{d}x.\]

Tích phân từng phần sẽ tìm được $\int_{0}^{1}\left(xe^x \right )^2\mathrm{d}x=\left.\frac{e^{2x}(2x^2-2x+1)}{4}\right|_0^1=\frac{e^2-1}{4}$, như vậy dấu bằng của bất đẳng thức trên xảy ra nên $f'(x)=k\cdot xe^x$ với $k$ là hằng số. Phần còn lại không khó.




#742265 A,B là 2 ma trận khả nghịch.Biết $A^{5}=I$,$AB^...

Đã gửi bởi nhungvienkimcuong on 27-11-2023 - 17:45 trong Đại số tuyến tính, Hình học giải tích

Cho A,B là hai ma trận khả nghịch. Giả sử $A^{5}=I$, $AB^{2}=BA$ và B#I. Tìm số nguyên k nhỏ nhất sao cho $B^{k}=I$, trong đó I là ma trận đơn vị.

 

Ta sẽ chứng minh rằng với số nguyên dương $n$ thỏa mãn $A^{-n}BA^n=B^{2^n}$ thì $A^{-n-1}BA^{n+1}=B^{2^{n+1}}$, thật vậy

\[B^{2^{n+1}}=\left(B^{2^n} \right )^2=\left(A^{-n}BA^n \right )^2=A^{-n}B^2A^n=A^{-n}(A^{-1}BA)A^n=A^{-n-1}BA^{n+1}.\]

Theo giả thiết thì $A^{-1}BA=B^2$, kết hợp với kết quả vừa chứng minh ta có

\[A^{-5}BA^5=B^{2^5}\implies B=B^{32}\implies B^{31}=I.\]

Với $k$ là số nguyên dương nhỏ nhất thỏa mãn $B^k=I$ thì $k\mid 31$, mà $k\neq 1$ nên $k=31$.




#742240 giải phương trình $x^{4}-1=3y^{2}$ với x,y nguy...

Đã gửi bởi nhungvienkimcuong on 26-11-2023 - 19:32 trong Số học

Chứng minh rằng phương trình sau không có nghiệm  nguyên dương:

$x^{4}-1=3y^{2}$

Bài này không hề tầm thường. Ljunggren đã chứng minh rằng phương trình $x^4-Dy^2=1$ có tối đa hai nghiệm, chứng minh của Cohn theo mình có thể hiểu kha khá dựa vào toán sơ cấp.




#742203 CMR: $a+b+c \geq ab + bc + ac$.

Đã gửi bởi nhungvienkimcuong on 24-11-2023 - 07:56 trong Bất đẳng thức và cực trị

Bài này có hướng giải theo cách lớp 9 không ạ
Cho $a, b, c$ là các số thực không âm thoả mãn:

$ab + bc + ac + abc\leq 4$. CMR: $$ab+bc+ac \leq a+b+c.$$

Theo nguyên lí Đi-rich-lê thì trong ba số $a-1,b-1,c-1$ phải có hai số cùng dấu; giả sử là $a-1$ và $b-1$. Khi đó

\[c(a-1)(b-1)\ge 0\implies c\ge ac+bc-abc\implies a+b+c\ge a+b-abc+ac+bc.\]

Như vậy ta cần chứng minh

\[a+b-abc+ac+bc\ge ab+bc+ca\iff a+b\ge ab(c+1).\]

Mặt khác theo giả thiết thì $c\le \frac{4-ab}{a+b+ab}$ (ở đây đang xét $a^2+b^2\neq 0$), do đó ta cần chứng tỏ

\[a+b\ge ab\left(\frac{4-ab}{a+b+ab}+1\right)\iff \frac{(a-b)^2}{a+b+ab}\ge 0.\]




#742153 $\left \{ x^2 \right \}+\left \{ y^2...

Đã gửi bởi nhungvienkimcuong on 17-11-2023 - 22:12 trong Số học

Chứng minh phương trình $\left \{ x^2 \right \}+\left \{ y^2 \right \}=\left \{ z^2 \right \}$ có vô số nghiệm trên tập $\mathbb{Q}\setminus \mathbb{Z}$

Gọi $a,b,c$ là các số nguyên dương thỏa mãn
\[2a^2<b^2\quad\text{và}\quad 2a^2+b^2=c^2,\tag{$\ast$}\]
khi đó $2\left(\frac{a}{b}\right)^2+1=\left(\frac{c}{b}\right)^2$. Như vậy với $x=y=\frac{a}{b}$ và $z=\frac{c}{b}$ thì phương trình $\{x^2\}+\{y^2\}=\{z^2\}$ có nghiệm trên $\mathbb{Q}\setminus \mathbb{Z}$.
Phần còn lại là chứng minh có vô số nghiệm, ta sẽ thực hiện điều đó bằng cách chứng tỏ rằng có vô hạn bộ ba số $a,b,c$ đôi một nguyên tố cùng nhau thỏa mãn $(\ast)$. Để ý đẳng thức
\[2(2mn)^2+(2m^2-n^2)^2=(2m^2+n^2)^2.\]
Từ đây ta chọn $a=2mn,b=2m^2-n^2$ và $c=2m^2+n^2$ với $m,n$ là hai số nguyên dương nguyên tố cùng nhau thỏa mãn $n<\frac{m}{2}$ và $n$ lẻ.
 
Ghi chú. Một bài toán khá tương tự như sau: Chứng minh rằng với mọi số nguyên $n\ge 2$ thì luôn tồn tại các số $x_1,x_2,\dots,x_n,x_{n+1}\in \mathbb{Q}\setminus \mathbb{Z}$ thỏa mãn
\[\left \{ x_1^3 \right \}+\left \{ x_2^3 \right \}+\dots+\left \{ x_n^3 \right \}=\left \{ x_{n+1}^3 \right \}.\]



#742139 tổng tất cả các ước dương của $n$ là $n(p-1)$ với $p...

Đã gửi bởi nhungvienkimcuong on 15-11-2023 - 20:45 trong Số học

Tìm số nguyên dương $n$ sao cho tổng tất cả các ước dương của $n$ (tính cả $n$) là $n(p-1)$ với $p$ là ước nguyên tố lớn nhất của $n$

Kí hiệu $\sigma(n)$ là tổng các ước dương của $n$, bài này sẽ được xử lí thông qua đánh giá sau:

\[\frac{\sigma(n)}{n}\le \text{Số ước nguyên tố của}\ n.\]

Giả sử phân tích thừa số nguyên tố của $n$ là $p_1^{a_1}p_2^{a_2}\dots p_k^{a_k}$ ($p_i$ là các số nguyên tố đôi một phân biệt, $a_i$ là các số nguyên dương), khi đó

\[\begin{align*} \frac{\sigma(n)}{n}=\prod_{i=1}^k\left ( 1+\frac{1}{p_i}+\dots+\frac{1}{p_i^{a_i}} \right )&<\prod_{i=1}^k\left ( 1+\frac{1}{p_i}+\dots+\frac{1}{p_i^{a_i}} +\cdots\right )\\&=\prod_{i=1}^k\frac{1}{1-\frac{1}{p_i}}<\prod_{i=1}^k\frac{1}{1-\frac{1}{i+1}}=k+1.\end{align*}\]

Với kết quả này thì phần còn lại không hề khó, kết hợp với giả thiết thì $p-1\le k$, mặt khác $p\ge 2k-1$ nên $k\le 2$. Từ đây bạn tự xử lí, chỉ cần xét $n\in \{2^x,3^x,2^x3^y\}$ rồi giải phương trình nghiệm nguyên.

 

Ghi chú. Một số bài toán nghiệm nguyên liên quan đến hàm số học có thể tham khảo tại đây, đâyđây.




#742048 Chứng minh rằng $d\vdots 30$.

Đã gửi bởi nhungvienkimcuong on 06-11-2023 - 09:36 trong Số học

Cho các số nguyên dương a, d sao cho: $a,a+d,a+2d,a+3d,a+4d$, đồng thời là các số nguyên tố. Chứng minh rằng $d\vdots 30$

Xem bài toán tổng quát hơn ở đây.




#742047 $(3^n-1) \vdots 2^{^{2023 }}$

Đã gửi bởi nhungvienkimcuong on 06-11-2023 - 09:32 trong Số học

Tìm số tự nhiên n nhỏ nhất sao cho

$(3^n-1) \vdots 2^{^{2023 }}$

Để tránh sử dụng bổ đề nâng lũy thừa thì làm như sau. Đặt $n=2^ab$ với $a,b$ là các số tự nhiên sao cho $b$ lẻ, khi đó

\[3^n-1=\big(3^{2^a}\big)^b-1=\left(3^{2^a}-1 \right )\bigg[\underbrace{\big(3^{2^a}\big)^{b-1}+\big(3^{2^a}\big)^{b-2}+\dots+3^{2^a}+1}_{B} \bigg].\]

Dễ thấy $B\equiv b\pmod{2}$, mà $b$ lẻ nên $B$ lẻ. Như vậy

\begin{equation}\label{1}2^{2023}\mid 3^n-1\iff 2^{2023}\mid 3^{2^a}-1.\end{equation}

Tiếp theo ta có

\[\begin{align*}3^{2^a}-1=( 3^{2^{a-1}}-1 )( 3^{2^{a-1}}+1)=\dots &=(3-1)(3+1)(3^2+1)\dots(3^{2^{a-1}}+1 )\\ &=8(3^2+1)(3^4+1)\dots(3^{2^{a-1}}+1 ).\end{align*}\]

Với mọi $k\in\{1,2,\dots,a-1\}$ thì $3^{2^k}+1$ là bội của $2$ nhưng không phải bội của $4$, do vậy $3^{2^a}-1$ là bội của $8\cdot 2^{a-1}=2^{a+2}$ nhưng không phải bội của $2^{a+3}$. Dẫn đến

\begin{equation}\label{2}2^{2023}\mid 3^{2^a}-1\iff a+2\ge 2023\iff a\ge 2021.\end{equation}

Từ \eqref{1} và \eqref{2} suy ra $n=2^{2021}$ là số nhỏ nhất cần tìm.




#742037 $ \frac{n}{x_1^2+x_2^2+x_3^2+\dots+ x_n^2}\leq\sum_{...

Đã gửi bởi nhungvienkimcuong on 05-11-2023 - 19:11 trong Bất đẳng thức và cực trị

Mời mọi người giải trí với bài tổng quát này nhé. 

Xét các số thực dương $x_1,x_2,x_3\ldots, x_n$ thỏa mãn $x_1x_2x_3\dots x_n=1$. Chứng minh rằng $$ \frac{n}{x_1^2+x_2^2+x_3^2+\dots+ x_n^2}\leq\sum_{cyc}\frac{1}{x_1^2+x_2+x_3+\dots+x_n}\leq \frac{n}{x_1+x_2+x_3+\dots+ x_n} $$

Vế trái dễ hơn chỉ cần sử dụng bất đẳng thức Cô-si Sơ-vác dạng mẫu, kết hợp với $\sum_{i=1}^nx_i^2\ge \sum_{i=1}^nx_i$.

Giờ đây tập trung vào vế phải, để cho dễ trình bày thì mình sẽ chứng minh với $n=3$ (các trường hợp khác tương tự). Ta có

\[\begin{align*} \sum\frac{1}{x_1^2+x_2+x_3}&=\sum\frac{1+x_2+x_3}{(x_1^2+x_2+x_3)(1+x_2+x_3)}\\&\le \sum\frac{1+x_2+x_3}{(x_1+x_2+x_3)^2}\\&=\frac{3+2(x_1+x_2+x_3)}{(x_1+x_2+x_3)^2}\\&\le \frac{x_1+x_2+x_3+2(x_1+x_2+x_3)}{(x_1+x_2+x_3)^2}\\&=\frac{3}{x_1+x_2+x_3}.\end{align*}\]




#742017 Chứng minh rằng $\frac{\left ( 2k \right )!...

Đã gửi bởi nhungvienkimcuong on 05-11-2023 - 12:06 trong Số học

Với các số nguyên $a,b$ nguyên tố cùng nhau, $a> b> 1$, ta xét dãy số sau: 

$u_n=\varphi (a^{2n-1}+b^{2n-1})$ với $n=1,2,3,...$ 

1. Chứng minh rằng nếu $p> 3$ là số nguyên tố lẻ và có số hạng nào đó của dãy trên bằng $2p$ thì $a+b=2p+1$ hoặc $a+b=2(2p+1)$

2. Chứng minh rằng $\frac{\left ( 2k \right )!}{k!}\mid \prod_{i=1}^{k}u_i$

Sử dụng kết quả này là giải quyết được.

Ngoài ra ở ý đầu tiên thì chú ý rằng nếu $\varphi(x)=2p$ thì $x\in \{q^k,2q^k\}$ với $q$ là số nguyên tố và $k$ là số nguyên dương, từ đây lập luận được rằng $k=1$ và $q=2p+1$.




#741988 gtnn $x+y+z$

Đã gửi bởi nhungvienkimcuong on 03-11-2023 - 04:05 trong Bất đẳng thức và cực trị

Cho $x,y,z$ thực thỏa mãn $3\leq x,y,z\leq 5$ và $x^2+y^2+z^2=50$

Tìm gtnn $x+y+z$

Cách 1.

Khai triển $(x-3)(y-3)(z-3)\ge 0$ ta có

\begin{equation*} xyz-3(xy+yz+zx)+9(x+y+z)-27\ge 0.\tag{1}\end{equation*}

Khai triển $(x-5)(y-5)(z-5)\le 0$ ta có

\begin{equation*}xyz-5(xy+yz+zx)+25(x+y+z)-125\le 0.\tag{2}\end{equation*}

Lấy $(1)$ trừ $(2)$ ta có được

\begin{equation*}2(xy+yz+zx)-16(x+y+z)+98\ge 0.\tag{3}\end{equation*}

Mặt khác $2(xy+yz+zx)=(x+y+z)^2-(x^2+y^2+z^2)=(x+y+z)^2-50$ nên $(3)$ tương đương

\[(x+y+z)^2-16(x+y+z)+48\ge 0\implies x+y+z\ge 12.\]

Dấu bằng xảy ra khi và chỉ khi $x=3,y=4,z=5$ cùng các hoán vị.

 

Cách 2.

Không mất tính tổng quát ta sắp xếp $x\ge y\ge z$. Giả sử

\[x+y+z<12\implies x+y<12-z\le 9.\]

Sử dụng khai triển Abel ta có

\begin{align*}x^2+y^2+z^2=x\cdot x+y\cdot y+z\cdot z&=x(x-y)+(x+y)(y-z)+(x+y+z)z\\&<5(x-y)+9(y-z)+12z\\&=5x+4y+3z\\&=x+4(x+y+z)-z\\&< 5+4\cdot 12-3 =50.\end{align*}
Từ mâu thuẫn này ta có $x+y+z\ge 12$.



#741951 $\lim_{n\to \infty }\sqrt[n]{1+cos(2n...

Đã gửi bởi nhungvienkimcuong on 31-10-2023 - 21:09 trong Giải tích

Tính giới hạn sau:

$\lim_{n\to \infty }\sqrt[n]{1+cos(2n)}$

Nhìn lướt qua thì cảm giác đây là bài toán bình thường cho phổ thông, đến lúc đặt bút thì chịu  :icon6: . Tìm trên mạng thấy xuất hiện ở đây.




#741916 $x_0=\frac{1}{2} , x_{n+1}=x_n+\...

Đã gửi bởi nhungvienkimcuong on 29-10-2023 - 20:49 trong Dãy số - Giới hạn

Bài toán:  Cho dãy số thực $(x_n)_n$ được định nghĩa như sau

 

$$x_0=\frac{1}{2} \, \, , \, \, x_{n+1}=x_n+\frac{x^{2}_{n}}{2017} \,\, , \,\, \forall n\in \mathbb{N}$$

 

Tìm số tự nhiên $k$ nhỏ nhất sao cho $x_k>1$

 

Note:  Đây là bài toán cũ không có lời giải của diễn đàn, thấy thú vị nên post lại cho các bạn giải thử. Mình thì tìm được $k=4034$, bạn nào tìm được giá trị nhỏ hơn thì cho lời giải nhé.

Dựa vào giả thiết thấy rằng hướng đi là cần tìm một sai phần phù hợp. Ta có

$$\frac{1}{x_{n+1}}=\frac{2017}{x_n(x_n+2017)} = \frac{1}{x_n}-\frac{1}{x_n+2017}.$$

Từ đây dẫn đến

$$2-\frac{1}{x_n}=\sum_{i=0}^{n-1}\frac{1}{x_i+2017}.$$

$\bullet$ Chứng minh $x_k<1$ với mọi $k\le 2017$.

Dễ thấy $x_i>0$ với mọi $i<k$, do vậy

\[2-\frac{1}{x_k}=\sum_{i=0}^{k-1}\frac{1}{x_i+2017}<\sum_{i=0}^{k-1}\frac{1}{2017}=\frac{k}{2017}\le 1.\]

$\bullet$ Chứng minh $x_{2018}>1$.

Vì $x_i<1$ với mọi $i<2018$ nên

\[2-\frac{1}{x_{2018}}=\sum_{i=0}^{2017}\frac{1}{x_i+2017}>\sum_{i=0}^{2017}\frac{1}{2018}= 1.\]

Vậy $k=2018$ là giá trị cần tìm (Đáp số được kiểm định bởi python).

n = 0
xn = 1/2

while xn < 1:
    xn = xn + xn ** 2 / 2017
    n += 1

print(n)



#741910 $e^{A+B}=e^A\,e^B=e^B\,e^A$

Đã gửi bởi nhungvienkimcuong on 29-10-2023 - 17:04 trong Đại số tuyến tính, Hình học giải tích

Mình từng làm bài này khi còn là sinh viên đại học. File gửi kèm  giaitichham.pdf   107.72K   55 Số lần tải

 




#741870 $x^{2021}+y!=y^{2021}+x!$

Đã gửi bởi nhungvienkimcuong on 27-10-2023 - 20:32 trong Số học

 Chứng minh rằng không tồn tại các số nguyên dương $x,y>2$ phân biệt sao cho:

$$x^{2021}+y!=y^{2021}+x!$$

Bài này ý tưởng rõ ràng, tuy nhiên lúc làm chi tiết cũng khá mắc công.

 

Đặt $m=2021$ và giả sử $x<y$. Lời giải xuất phát từ nhận xét: xét $p$ là ước nguyên tố bất kì của $x$, từ giả thiết suy ra $p\mid y$, do vậy

\begin{equation}\label{a} m\le v_p(y^m-x^m)=v_p(y!-x!)=v_p(x!)<\frac{x}{p-1}.\end{equation} 

Mệnh đề

Xét hàm số $f\colon \mathbb{N}^*\to \mathbb{Z}$ được xác định bởi $f(k)=k^m-k!$. Khi đó hàm $f$ nhận giá trị âm và giảm ngặt trong khoảng $(2m-1,+\infty)$.

Xét trường hợp $p\ge 3$, từ \eqref{a} ta có $x>2m$, kết hợp với Theorem suy ra $f(x)> f(y)$.

Vậy chỉ còn trường hợp $x$ là lũy thừa của $2$, cũng từ \eqref{a} thì $x>m$. Đương nhiên $y$ chẵn, xét $q$ là một ước nguyên tố lẻ của $y$.

  • Nếu $q\le x$, từ giả thiết suy ra $q\mid x$ (vô lí).
  • Với $q>x$ suy ra $y\ge 2q>2x>2m$, theo Theorem thì $f(y)<0$. Từ đây dễ thấy $f(x)$ âm hay dương cũng vô lí (trường hợp âm kết hợp với phần chứng minh của Theorem).

 

 

Ghi chú. Một số bài toán tương tự có thể kể đến như sau

Bài 1 (Trung Âu 2015). Tìm tất cả cặp số nguyên dương $(a,b)$ thỏa mãn

$$a!+b!=a^b+b^a.$$

Bài 2. Tìm tất cả các số nguyên dương $n$ sao cho $(n-1)!$ là bội của $n^2$.

Bài 3 (IMO 2019). Tìm tất cả các cặp số nguyên dương $(k,n)$ sao cho

$$k!=(2^n-1)(2^n-2)(2^n-4)\cdots (2^n-2^{n-1}).$$




#741846 $\frac{m_a}{h_{a}}+\frac{m_...

Đã gửi bởi nhungvienkimcuong on 26-10-2023 - 20:37 trong Hình học

 

Trong  tam giác $ABC$ nhọn có $h_{a}, h_{b},h_{c}$ là độ dài các đường cao xuất phát từ các đỉnh $A,B,C$.  $m_{a},m_{b},m_{c}$, là độ dài các đường trung tuyến hạ từ các đỉnh $A,B,C$.  $(O;R),  (O';r)$ là tâm đường tròn ngoại tiếp và nội tiếp tam giác $ABC$. CMR : $\frac{m_{a}}{h_{a}}+\frac{m_{b}}{h_{b}}+\frac{m_{c}}{h_{c}}\leq \frac{R+r}{r}$

Gọi $A',B',C'$ lần lượt là trung điểm các đoạn $BC,CA,AB$. Kí hiệu $S$ là diện tích của tam giác $ABC$, chú ý rằng

\[m_a=AA'\le OA+OA'=R+OA',\quad \frac{(a+b+c)r}{2}=\frac{a\cdot OA'+b\cdot OB'+c\cdot OC'}{2}=S.\]

Do vậy

\[\sum\frac{m_a}{h_a}=\sum\frac{am_a}{ah_a}\le \sum\frac{a(R+OA')}{2S}=R\frac{\sum a}{2S}+\frac{\sum a\cdot OA'}{2S}=\frac{R}{r}+1.\]

Screenshot 2023-10-26 223602.png




#741783 Tìm a,b nguyên dương thỏa mãn $a^{4}+10a^{2}+2^...

Đã gửi bởi nhungvienkimcuong on 18-10-2023 - 18:26 trong Số học

Tìm $a,b \in Z , a,b>0$ thỏa mãn $a^{4}+10a^{2}+2^{b}$ là 1 số chính phương

Vì bài này để ở THCS nên mình sẽ nêu lại một số ý cần thiết liên quan tới số mũ đúng. Tài liệu tham khảo: Phạm Quang ToànAmir Hosein.

Mệnh đề

Với mỗi số nguyên dương $x$, kí hiệu $v_2(x)$ là số tự nhiên $k$ lớn nhất sao cho $2^k\mid x$ (ví dụ $v_2(12)=2$ và $v_2(15)=0$). Với $x,y$ là các số nguyên dương thì ta có

  1. $v_2(x^y)=y\cdot v_2(x)$, do vậy $v_2(x^2)$ luôn là số chẵn.
  2. $v_2(x+y)= \min\big(v_2(x),v_2(y)\big)$ nếu $v_2(x)\neq v_2(y)$.

 

Quay lại bài toán. Xét $b\ge 5$, nếu $a$ lẻ thì 

\[a^2\equiv 1\pmod{4}\implies a^4+10a^2+2^b\equiv 3\pmod{4}.\]

Điều này vô lí thì số chính phương khi chia cho $4$ chỉ dư $0$ hoặc $1$, do vậy $a$ chẵn. Nếu $b\ge 2v_2(a)+2$ thì theo Mệnh đề 2 ta có

\[v_2(a^4+10a^2+2^b)=v_2(10a^2)=2v_2(a)+1,\]

vô lí theo Mệnh đề 1. Vậy $b\le 2v_2(a)+1$, suy ra

\[a^4+10a^2+2^b\le a^4+12a^2<(a^2+6)^2.\]

Mặt khác $b\ge 5$ nên $a^4+10a^2+2^b>(a^2+5)^2$. Vậy $b\ge 5$ thì không tồn tại $a$, còn lại chỉ cần xét $b\in\{1,2,3,4\}$ thì đơn giản rồi.




#741779 $ \sqrt{a^2+16bc} + \sqrt{b^2+16ca} +...

Đã gửi bởi nhungvienkimcuong on 18-10-2023 - 11:17 trong Bất đẳng thức - Cực trị

Cho $a,b,c \ge 0 thoả ab+bc+ca>0$.Chứng minh rằng:
$ \sqrt{a^2+16bc} + \sqrt{b^2+16ca} + \sqrt{c^2+16ab} \ge 6\sqrt{ab+bc+ca} $

Cơ bản thì lời giải của mình khá giống anh Quý, khác là mình may mắn tìm được số $24$ ở \eqref{ab} nên có phần ngắn hơn.

 

Về mặt ý tưởng, ta mong muốn làm giảm số căn thức lại bằng cách sử dụng một bất đẳng thức phụ. Giả sử $c=\min(a,b,c)$, mình tìm được kết quả sau

\begin{equation}\label{ab}\sqrt{a^2+16bc}+\sqrt{b^2+16ca}\ge \sqrt{(a+b)^2+{\color{Red} 24}(a+b)c}.\end{equation}

Áp dụng \eqref{ab} kết hợp với $\sqrt{c^2+16ab}\ge 4\sqrt{ab}$, do vậy cần chứng minh

\[\sqrt{(a+b)^2+24(a+b)c}\ge 6\sqrt{ab+bc+ca}-4\sqrt{ab}.\]

Bình phương hai vế tương đương với

\begin{equation}(a+b)^2+48\sqrt{ab(ab+bc+ca)}\ge 52ab+12c(a+b).\end{equation}

Ta thấy rằng $ab(ab+bc+ca)-\left(ab+\frac{c(a+b)}{3}\right)^2=\dfrac{c(a+b)(3ab-ac-bc)}{9} \geq 0$Do vậy

\[VT(2)\ge 4ab+48\left(ab+\frac{c(a+b)}{3} \right )=52ab+16c(a+b)\ge VP(2).\]

 

 

Ghi chú. Một số bài toán có vẻ tương tự ở đâyđây.




#741736 Công thức tính mode của mẫu dữ liệu ghép nhóm

Đã gửi bởi nhungvienkimcuong on 15-10-2023 - 16:34 trong Xác suất - Thống kê

Cố gắng chứng minh công thức $\eqref{1}$ bằng hình học, ta thấy rằng, hợp lý nhất thì mode phải là hoành độ của giao điểm $D$ của $AC$ và $FH$. Đặt $x=CE$.

attachicon.gif screenshot_1697216271.png

Việc lấy giao điểm $AC\cap FH$ là chưa chính xác, vì điểm đạt được mode phải thỏa mãn "tỉ lệ". Như vậy để thu được kết quả đúng thì cần lấy giao điểm $BF\cap CG$.

 

Nếu coi mẫu dữ liệu là liên tục thì mode là điểm cực đại của hàm phân phối $F(x)$. Bài toán trở thành:
Cho hàm số $F(x)$ liên tục trên $[a_1;a_4]$. Chia đoạn $[a_1;a_4]$ thành ba đoạn bằng nhau $[a_1; a_2], [a_2; a_3]; [a_3; a_4]$. Đặt $$\sum_{x=a_i}^{a_{i+1}}F(x)=m_i, \quad i = 1,2,3.$$
Biết rằng $m_2>m_1; m_2>m_3$. Tìm điểm cực đại của $F(x)$.

Điểm cực đại sẽ là nghiệm của phương trình $F'(x)=0$.




#741706 Bài toán về giới hạn trong đề thi HSG cấp trường

Đã gửi bởi nhungvienkimcuong on 13-10-2023 - 05:49 trong Dãy số - Giới hạn

Còn câu a thì sao ạ??????

Từ ý đầu tiên của câu a nên mới có dự đoán $u_n>n^2$, ngoài ra $n<\sqrt{u_n}<n+1$ nên $\left \lfloor \sqrt{u_n} \right \rfloor=n$.




#741698 Bài toán về giới hạn trong đề thi HSG cấp trường

Đã gửi bởi nhungvienkimcuong on 12-10-2023 - 19:55 trong Dãy số - Giới hạn

Gợi ý: chứng minh $u_n>n^2$ bằng phương pháp quy nạp, từ đó suy ra $u_n<(n+1)^2$. Như vậy $n^2<u_n<(n+1)^2$.




#741675 GTNN của $A= \frac{1}{a^{2}+b^{2...

Đã gửi bởi nhungvienkimcuong on 09-10-2023 - 22:26 trong Bất đẳng thức và cực trị

Tìm GTNN của A biết $a,b,c \ge 0: a+b+c \le 3$ và $A= \frac{1}{a^{2}+b^{2}}+\frac{1}{b^{2}+c^{2}}+\frac{1}{c^{2}+a^{2}}$

Dấu bằng xảy ra khi một biến bằng $0$ và hai biến còn lại bằng nhau, ta sẽ chứng minh

\[ \frac{1}{a^{2}+b^{2}}+\frac{1}{b^{2}+c^{2}}+\frac{1}{c^{2}+a^{2}}\ge\frac{10}{(a+b+c)^2}.\]

Ý tưởng bài này là giả sử $c=\min(a,b,c)$, khi đó

\[a^2+b^2\le \left(a+\frac{c}{2}\right)^2+\left(b+\frac{c}{2}\right)^2,\quad b^2+c^2\le \left(b+\frac{c}{2}\right)^2,\quad c^2+a^2\le \left(a+\frac{c}{2}\right)^2.\]

Phần còn lại chỉ cần chứng minh $\frac{1}{x^2+y^2}+\frac{1}{y^2}+\frac{1}{x^2}\ge \frac{10}{(x+y)^2}$ với $x=a+\frac{c}{2}$ và $y=b+\frac{c}{2}$. Bài toán hai biến này thì không quá khó, bạn tự giải quyết nhé.

 

 

Ghi chú. Một số bài toán hoàn toàn tương tự, tham khảo ở đâyđây.




#741669 $u_1=1; u_{n+1}=u_n+\frac{1}{2u_n} ,...

Đã gửi bởi nhungvienkimcuong on 09-10-2023 - 11:30 trong Dãy số - Giới hạn

Cho dãy số thực $(u_n)$, $n\in N^{*}$ xác định bởi $u_1=1; u_{n+1}=u_n+\frac{1}{2u_n} , n\in N^{*}$. Tính $lim \frac{u_n}{\sqrt{n}}$ và tìm $[u_{2023}]$ 

Ý tưởng tương tự ở đây.




#741651 $a_{n+1}=\sqrt{2-\sqrt{4-a_{n}^...

Đã gửi bởi nhungvienkimcuong on 07-10-2023 - 18:14 trong Dãy số - Giới hạn

Cho $a_{0}=\sqrt{2}$,$b_{0}=2$,$a_{n+1}=\sqrt{2-\sqrt{4-a_{n}^{2}}}$,$b_{n+1}=\frac{2b_{n}}{2+\sqrt{4+b_{n}^{2}}}$

1. Chứng minh $(a_{n},b_{n})$ không tăng và hội tụ về 0

2. Chứng minh $(2^{n}a_{n})$ tăng và $(2^{n}b_{n})$ giảm và hai dãy đó hội tụ về cùng một giá trị

3. Chứng minh rằng tồn tại một số C sao cho với mọi n thì bất đẳng thức sau luôn đúng:

$0<b_{n}-a_{n}<\frac{C}{8^{n}}$

Với hình thức truy hồi như này thì nghĩ ngay đến công thức lượng giác, mò một tí ta thấy rằng với $0<\alpha<\frac{\pi}{2}$ thì

\[\sqrt{2-\sqrt{4-(2\sin \alpha)^2}}=2\sin\frac{\alpha}{2},\quad \frac{2\cdot 2\tan\alpha}{2+\sqrt{4+(2\tan\alpha)^2}}=2\tan\frac{\alpha}{2}.\]

Từ đây quy nạp thu được $a_n=2\sin\frac{\pi}{2^{n+2}}$ và $b_n=2\tan\frac{\pi}{2^{n+2}}$.

 

Có được công thức của hai dãy thì dễ dàng xử lí hai ý đầu tiên, ý còn lại tương đương với $0<\frac{\tan\frac{\pi}{2^{n+2}}-\sin\frac{\pi}{2^{n+2}}}{\left ( \frac{\pi}{2^{n+2}} \right )^3}<C'$. Chú ý rằng

\[\lim_{n\to \infty}\frac{\tan\frac{\pi}{2^{n+2}}-\sin\frac{\pi}{2^{n+2}}}{\left ( \frac{\pi}{2^{n+2}} \right )^3}=\lim_{x\to 0}\frac{\tan x-\sin x}{x^3}=\frac{1}{2}\]

là giải quyết xong.